0 Daumen
177 Aufrufe

Aufgabe:

Ich soll die Menge der \( z \in \mathbb{C} \) bestimmen für die die Reihe konvergiert.

a) \( \sum \limits_{n \geq 0}\left(\frac{1}{2}+\frac{1}{2 n+1}\right)^{n} z^{2 n} \)

b) \( \sum \limits_{n \geq 0} 2^{-n} 3^{(-1)^{n} n} z^{3 n} \)


Problem/Ansatz:

Ich verstehe einfach nicht wie ich hier solche Aufgaben lösen soll. Habe stundenlang es mithilfe der Vorlesungsskripts probiert oder mithilfe von Wikipedia, aber ich ich verstehe es einfach von vorne bis hinten nicht.

Avatar von

1 Antwort

0 Daumen
 
Beste Antwort

Hallo

du hast sicher die 2 Methoden den Konvergenzradius zu bestimmen im Skript  Wurzel und Quotient was ist bei a wohl nützlicher?, schreib z^3n=(z^3)^n und bestimme den Konvergenzradius für z^3 ( bzw, für z^2) daraus dann den für z, ich nehme an,, das (-1)^n steht nicht im Exponenten sondern so dass bei 2 die Summanden (-1)^n*3^n/2^n *(z^3)^n sind? dann hast du eine Leibnizreihe und die Summanden müssen eine Nullfolge bilden

Dann schreib was du mit den Tips erreichst oder wo es noch hakt.

Gruß lul

Avatar von 107 k 🚀

Ein anderes Problem?

Stell deine Frage

Willkommen bei der Mathelounge! Stell deine Frage einfach und kostenlos

x
Made by a lovely community